Problem involving a rotating frame. Help

In summary, the rotating frame appears to give a centripetal force that's not correct. The centrifugal and Coriolis forces both add up to give the correct force, but it's not clear why the centrifugal force is supposed to be radially outward instead of inward like the Coriolis force.
  • #1
ashishsinghal
462
0
This is a conceptual problem which I am facing for many days.

If we convert a scenario in an inertial frame into a rotating frame, we apply a pseudo force i.e. centrifugal force radially outwards on the particle. Right?

Also if a particle is having a circular motion in any frame of reference it must have a centripetal force directed radially inwards in that frame. Right?

Now, if I have a particle A which is at rest with respect to the ground, it has no force on it with respect to the ground. Now if I see this particle in a frame of reference rotating about origin O, I will apply centrifugal force directed along OA. But since in this frame A seems to undergo circular motion it must have an acceleration directed along AO in the rotating frame. But this is not happening. Why?

Please help ASAP. If I am not able to show the situation clearly please tell me.
 
Physics news on Phys.org
  • #2
ashishsinghal said:
If we convert a scenario in an inertial frame into a rotating frame, we apply a pseudo force i.e. centrifugal force radially outwards on the particle. Right?
Right.

Also if a particle is having a circular motion in any frame of reference it must have a centripetal force directed radially inwards in that frame. Right?
Right.

Now, if I have a particle A which is at rest with respect to the ground, it has no force on it with respect to the ground. Now if I see this particle in a frame of reference rotating about origin O, I will apply centrifugal force directed along OA. But since in this frame A seems to undergo circular motion it must have an acceleration directed along AO in the rotating frame. But this is not happening. Why?
If it's going in a circle (in that frame) it certainly has a centripetal acceleration.

Realize that the outward centrifugal force is not the only pseudo-force in this scenario. There is also a Coriolis force. Combined, those two give a net force (pseudo-force) in the centripetal direction, as required.

Does that get at your concerns?
 
  • #3
Even I thought the same way. But the coriolis force is not directed radially. It is a tangential force. Also coriolis force depends upon the rate of change of radial distance which here is zero.
 
  • #4
Coriolis force is the tangential force experienced on the particle which is coming closer or going away from the center of circular motion. Just like the winds change directions.
 
  • #5
ashishsinghal said:
Even I thought the same way. But the coriolis force is not directed radially. It is a tangential force. Also coriolis force depends upon the rate of change of radial distance which here is zero.
The Coriolis force will be radially inward.
 
  • #6
Can you please explain why? As according to what I have read it is tangential.
 
  • #7
There is no centripetal force in this case.
Since in the rotating frame, the object appears to be moving circularly, sum of all forces should be equal to mv^2/r
The forces acting on the object here is the pseudo force due to the rotation of my frame.
thus, Centrifugal force=mv^2/r
Does this solve your problem?
 
  • #8
ashishsinghal said:
Can you please explain why? As according to what I have read it is tangential.
I have no idea why you think that the Coriolis force is tangential. It is always perpendicular to the velocity. It's given by [tex]-2m\vec{\Omega}\times\vec{v}[/tex]
where Ω is the angular velocity vector of the rotating frame. (See Coriolis force or any classical mechanics textbook.)

Thus the Coriolis force is radially inward and equal to 2mω2r and the centrifugal force is radially outward and equal to mω2r. Thus the net force is inward and equal to mω2r, just as you would expect for circular motion.
 
  • #9
Aniket1 said:
There is no centripetal force in this case.
Since in the rotating frame, the object appears to be moving circularly, sum of all forces should be equal to mv^2/r
There must be a net centripetal force, acting radially inward.
The forces acting on the object here is the pseudo force due to the rotation of my frame.
thus, Centrifugal force=mv^2/r
The centrifugal force acts radially outward. So that's not enough to explain things. See my last post about the Coriolis force.
 
  • #10
There is nothing very complicated involved. For a free particle in the inertial frame with cartesian coordinates [itex]x_j[/itex] you have the Lagrangian
[tex]L=\frac{m}{2} \dot{\vec{x}}^2=\frac{m}{2} \dot{x}_j \dot{x}_j.[/tex]
Here and in the following Einstein summation is implied.

In terms of the coordinates [itex]y_j[/itex] fixed in the rotating frame (let's assume the rotation is around the three-axis) you have
[tex]\begin{pmatrix}
x_1 \\ x_2 \\ x_3 \end{pmatrix}=\begin{pmatrix}
\cos (\omega t) & -\sin (\omega t) & 0\\
\sin (\omega t) & \cos (\omega t) & 0 \\
0 & 0 & 1
\end{pmatrix} \begin{pmatrix} y_1 \\ y_2 \\y_3 \end{pmatrix} \qquad (*). [/tex]

Now take the time derivative of this and pluck it into the above Lagrangian. You'll get after some algebra
[tex]L=\frac{m}{2} \left [\dot{y}_j \dot{y}_j+2 \omega (y_1 \dot{y}_2-y_2 \dot{y}_1)+\omega^2 (y_1^2+y_2^2) \right ].[/tex]

With [itex]\vec{\omega}=\omega (0,0,1)^t[/itex] this reads
[tex]L=\frac{m}{2} \left [\dot{\vec{y}}^2+2 \vec{\omega} \cdot (\vec{y} \times \dot{\vec{y}}) + (\vec{\omega} \times \vec{y})^2 \right].[/tex]

With the Euler-Lagrange equations you'll get the correct equations of motion, which can be reinterpreted as the motion of a particle under the influence of the Coriolis and the centrifugal force:
[tex]m \ddot{\vec{y}}=-m \left [2 \vec{\omega} \times \dot{\vec{y}}-\vec{\omega} \times (\vec{\omega} \times \vec{y}) \right ].[/tex]

Of course you could also simply derive Eq. (*) two times wrt. to time, but that's more cumbersome :-).
 
  • #11
vanhees71 said:
There is nothing very complicated involved.

Sir, thanks for all the effort but after the work you just showed me I would seriously ask you to consider your definition of complicated. But thanks for your effort.
 
  • #12
Doc Al said:
I have no idea why you think that the Coriolis force is tangential. It is always perpendicular to the velocity. It's given by [tex]-2m\vec{\Omega}\times\vec{v}[/tex]
where Ω is the angular velocity vector of the rotating frame. (See Coriolis force or any classical mechanics textbook.)

Thus the Coriolis force is radially inward and equal to 2mω2r and the centrifugal force is radially outward and equal to mω2r. Thus the net force is inward and equal to mω2r, just as you would expect for circular motion.

I went to the wikipedia link you posted. Yup,I admit I was mistaken. Coriolis force can be in any direction in the plane of circular motion. Thanks a lot.
 
  • #13
Thanks everyone. Special thanks to Doc Al. This thread is closed.
 

1. What is a rotating frame?

A rotating frame is a reference frame that is constantly rotating, meaning that the axes of the frame are changing direction and magnitude over time.

2. How does a rotating frame affect physical phenomena?

A rotating frame can introduce fictitious forces, such as the Coriolis force and the centrifugal force, that affect the motion of objects within the frame.

3. What is the Coriolis force in a rotating frame?

The Coriolis force is a fictitious force that appears to act on objects in a rotating frame due to their inertia. It is responsible for the deflection of objects moving in a straight path in the rotating frame.

4. How is the Coriolis force related to the rotation of the Earth?

The rotation of the Earth creates a rotating frame, and the Coriolis force is a result of this rotation. It is responsible for the rotation of large-scale weather patterns and the direction of ocean currents.

5. Can the Coriolis force be ignored in certain situations?

Yes, the Coriolis force is typically negligible for small-scale motions and short time periods. It becomes more significant for larger distances and longer time periods, such as in weather systems or on a planetary scale.

Similar threads

  • Mechanics
Replies
19
Views
2K
Replies
15
Views
844
Replies
2
Views
732
Replies
25
Views
1K
Replies
17
Views
1K
Replies
17
Views
2K
Replies
5
Views
740
Replies
37
Views
3K
Replies
2
Views
964
Replies
2
Views
1K
Back
Top